Mathcenter Forum  

Go Back   Mathcenter Forum > คณิตศาสตร์ทั่วไป > ปัญหาคณิตศาสตร์ทั่วไป
สมัครสมาชิก คู่มือการใช้ รายชื่อสมาชิก ปฏิทิน ข้อความวันนี้

ตั้งหัวข้อใหม่ Reply
 
เครื่องมือของหัวข้อ ค้นหาในหัวข้อนี้
  #1  
Old 08 มกราคม 2005, 21:58
nooonuii nooonuii ไม่อยู่ในระบบ
ผู้พิทักษ์กฎทั่วไป
 
วันที่สมัครสมาชิก: 25 พฤษภาคม 2001
ข้อความ: 6,408
nooonuii is on a distinguished road
Post โจทย์ปัญหาคณิตศาสตร์ สวัสดีปีใหม่ 2548 ภาคสอง

เห็นว่ากระทู้เก่าเริ่มโหลดช้าแล้วก็เลยขอมาต่อกระทู้ใหม่ดีกว่าครับ

21. จงหาค่าของ \[ \sum \frac{\cos {n\theta}}{3^{n}} \text{เมื่อ} \cos {\theta} = 1/4 \]
__________________
site:mathcenter.net คำค้น

08 มกราคม 2005 22:01 : ข้อความนี้ถูกแก้ไขแล้ว 1 ครั้ง, ครั้งล่าสุดโดยคุณ nooonuii
ตอบพร้อมอ้างอิงข้อความนี้
  #2  
Old 08 มกราคม 2005, 22:04
aaaa's Avatar
aaaa aaaa ไม่อยู่ในระบบ
ลมปราณบริสุทธิ์
 
วันที่สมัครสมาชิก: 01 มกราคม 2005
ข้อความ: 109
aaaa is on a distinguished road
Post

คุณ nooonuii ลองกลับไปดูข้อ 20 อีกครั้งครับ น่าจะมีปัญหา
ตอบพร้อมอ้างอิงข้อความนี้
  #3  
Old 08 มกราคม 2005, 22:11
aaaa's Avatar
aaaa aaaa ไม่อยู่ในระบบ
ลมปราณบริสุทธิ์
 
วันที่สมัครสมาชิก: 01 มกราคม 2005
ข้อความ: 109
aaaa is on a distinguished road
Post

เฉลยข้อ 21
ได้เท่ากับ
\[
\text{Re}\;\!\left\{\sum_{m=0}^\infty\frac{e^{in\theta}}{3^n}\right\}=\text{Re}\;\!\left\{\frac{1}{1-e^{i\theta}/3}\right\}=\frac{33}{34}
\]
ตอบพร้อมอ้างอิงข้อความนี้
  #4  
Old 08 มกราคม 2005, 22:49
aaaa's Avatar
aaaa aaaa ไม่อยู่ในระบบ
ลมปราณบริสุทธิ์
 
วันที่สมัครสมาชิก: 01 มกราคม 2005
ข้อความ: 109
aaaa is on a distinguished road
Post

22. จงหาค่าสูงสุดของ \( 2^{-x}+2^{-1/x} \) โดย \( x>0 \)

08 มกราคม 2005 23:04 : ข้อความนี้ถูกแก้ไขแล้ว 2 ครั้ง, ครั้งล่าสุดโดยคุณ aaaa
ตอบพร้อมอ้างอิงข้อความนี้
  #5  
Old 08 มกราคม 2005, 23:37
gon's Avatar
gon gon ไม่อยู่ในระบบ
ผู้พิทักษ์กฎขั้นสูง
 
วันที่สมัครสมาชิก: 29 มีนาคม 2001
ข้อความ: 4,608
gon is on a distinguished road
Post

ข้อที่ 21 ผมหามาได้แบบนี้

\[ \sum_{n=1}^{\infty}r^ncosn\theta = \frac{rcos\theta-r^2}{1-2rcos\theta+r^2} \]

เมื่อแทน \( r = \frac{1}{3}, cos\theta = \frac{1}{4}\) ก็จะได้ \( -\frac{1}{34} \) รู้สึกว่ามันจะกลับข้างกับคุณ aaaa เลยนะครับ. คุณ aaaa หาผลบวกจาก \( n = 1 \)หรือเปล่า ? หรือว่าผมหาผิดตรงไหน
ตอบพร้อมอ้างอิงข้อความนี้
  #6  
Old 08 มกราคม 2005, 23:39
nooonuii nooonuii ไม่อยู่ในระบบ
ผู้พิทักษ์กฎทั่วไป
 
วันที่สมัครสมาชิก: 25 พฤษภาคม 2001
ข้อความ: 6,408
nooonuii is on a distinguished road
Post

ถูกทั้งสองคนนั่นแหละครับ พี่กรเริ่มจาก n=1 ครับ
__________________
site:mathcenter.net คำค้น
ตอบพร้อมอ้างอิงข้อความนี้
  #7  
Old 09 มกราคม 2005, 00:00
aaaa's Avatar
aaaa aaaa ไม่อยู่ในระบบ
ลมปราณบริสุทธิ์
 
วันที่สมัครสมาชิก: 01 มกราคม 2005
ข้อความ: 109
aaaa is on a distinguished road
Post

ข้อ 23. กำหนดให้ \( \{a_n\} \) เป็นลำดับของจำนวนจริงบวกซึ่ง \( \displaystyle\lim_{n\to\infty}a_n=0 \) จงพิสูจน์ว่า
\[
\lim_{n\to\infty}\frac{a_1+\sqrt{2}a_2+\cdots+\sqrt{n}a_n}{1+\sqrt{2}+\cdots+\sqrt{n}}=0
\]
ตอบพร้อมอ้างอิงข้อความนี้
  #8  
Old 09 มกราคม 2005, 00:16
gon's Avatar
gon gon ไม่อยู่ในระบบ
ผู้พิทักษ์กฎขั้นสูง
 
วันที่สมัครสมาชิก: 29 มีนาคม 2001
ข้อความ: 4,608
gon is on a distinguished road
Smile

ข้อ 22 ผมใช้แคลครับ. นั่งมองอสมการแต่ไม่รู้จะเอาอะไรมาใช้ดี ตอนนี้ไม่ชำนาญเลย ก็จะได้เงื่อนไขคือ \[ x^2 = 2^{x-\frac{1}{x}} \Rightarrow x = 1 \] เมื่อลองหา \( f"(1) = (ln2)(ln 2 - 2) < 0 \Rightarrow f(1) = \frac{1}{2} + \frac{1}{2} = 1\) เป็นค่าสูงสุด ถูกหรือเปล่าครับ. ?
ตอบพร้อมอ้างอิงข้อความนี้
  #9  
Old 09 มกราคม 2005, 00:40
nooonuii nooonuii ไม่อยู่ในระบบ
ผู้พิทักษ์กฎทั่วไป
 
วันที่สมัครสมาชิก: 25 พฤษภาคม 2001
ข้อความ: 6,408
nooonuii is on a distinguished road
Post

ข้อ 22 ตอบ 1 ครับ

ก่อนอื่นใช้แคลคูลัสพิสูจน์ว่า \( \large{2^{x} \geq 2x} \) ทุกค่า x > 1

ดังนั้น ถ้า x>1 จะได้ว่า

\( \large {(1-\frac{1}{2^{x}})^{x} > 1-\frac{x}{2^{x}} \geq \frac{1}{2}} \)

โดยอสมการของ Bernoulli

ถ้า 0<x<1 จะได้ว่า 1/x > 1 ก็สามารถใช้อสมการข้างบนได้

\ \( \large{ \frac{1}{2^{x}} + \frac{1}{2^{1/x}} \leq 1} \) ทุกค่า x>0 และสมการเป็นจริงเมื่อ x=1
__________________
site:mathcenter.net คำค้น

09 มกราคม 2005 00:49 : ข้อความนี้ถูกแก้ไขแล้ว 2 ครั้ง, ครั้งล่าสุดโดยคุณ nooonuii
ตอบพร้อมอ้างอิงข้อความนี้
  #10  
Old 09 มกราคม 2005, 00:47
gon's Avatar
gon gon ไม่อยู่ในระบบ
ผู้พิทักษ์กฎขั้นสูง
 
วันที่สมัครสมาชิก: 29 มีนาคม 2001
ข้อความ: 4,608
gon is on a distinguished road
Post

ข้อ 23 ทำแบบนี้ได้หรือเปล่าครับ.

\[ \frac{a_1+a_2+\cdots a_n}{1+\sqrt{2}+\cdots\sqrt{n}}
\leq \frac{a_1+\sqrt{2}a_2+\cdots+\sqrt{n}a_n}{1+\sqrt{2}+\cdots+\sqrt{n}} \]
\[ \leq \frac{\sqrt{n}a_1+\sqrt{n}a_2+\cdots+\sqrt{n}a_n}{1+\sqrt{2}+\cdots+\sqrt{n}} = \frac{\sqrt{n}}{1+\sqrt{2}+\cdots\sqrt{n}}(a_1+a_2+\cdots a_n)
\]

ยกมาคนละบรรทัดดีกว่า. อ่านแล้วไม่สะดุดบรรทัด

09 มกราคม 2005 01:07 : ข้อความนี้ถูกแก้ไขแล้ว 1 ครั้ง, ครั้งล่าสุดโดยคุณ gon
ตอบพร้อมอ้างอิงข้อความนี้
  #11  
Old 09 มกราคม 2005, 00:53
aaaa's Avatar
aaaa aaaa ไม่อยู่ในระบบ
ลมปราณบริสุทธิ์
 
วันที่สมัครสมาชิก: 01 มกราคม 2005
ข้อความ: 109
aaaa is on a distinguished road
Post

แล้วยังไงต่อเหรอครับ
ตอบพร้อมอ้างอิงข้อความนี้
  #12  
Old 09 มกราคม 2005, 00:59
gon's Avatar
gon gon ไม่อยู่ในระบบ
ผู้พิทักษ์กฎขั้นสูง
 
วันที่สมัครสมาชิก: 29 มีนาคม 2001
ข้อความ: 4,608
gon is on a distinguished road
Smile

แล้วก็ใช้ Squeeze Theorem อะไรนั่น ต่อ คือ ลิมิตทางซ้ายกับขวาเป็นศูนย์ทั้งคู่ (ใช่หรือเปล่าหว่า) เอ๊ะทางขวามันศูนย์ไหม ?

09 มกราคม 2005 01:03 : ข้อความนี้ถูกแก้ไขแล้ว 1 ครั้ง, ครั้งล่าสุดโดยคุณ gon
ตอบพร้อมอ้างอิงข้อความนี้
  #13  
Old 09 มกราคม 2005, 01:05
gon's Avatar
gon gon ไม่อยู่ในระบบ
ผู้พิทักษ์กฎขั้นสูง
 
วันที่สมัครสมาชิก: 29 มีนาคม 2001
ข้อความ: 4,608
gon is on a distinguished road
Icon22

ท่าจะมั่วครับ. ขอนอนก่อนล่ะ พรุ่งนี้ต้องออกแต่เช้าเลย
ตอบพร้อมอ้างอิงข้อความนี้
  #14  
Old 09 มกราคม 2005, 01:26
nooonuii nooonuii ไม่อยู่ในระบบ
ผู้พิทักษ์กฎทั่วไป
 
วันที่สมัครสมาชิก: 25 พฤษภาคม 2001
ข้อความ: 6,408
nooonuii is on a distinguished road
Post

24. ให้ \( \large{a_{1} + a_{2} + ...} \) เป็นอนุกรมบวกที่ลู่เข้า จงพิสูจน์ว่า

\[ \large{ (1+a_{1})(1+a_{2}) \dots } \] ลู่เข้า
__________________
site:mathcenter.net คำค้น

09 มกราคม 2005 01:29 : ข้อความนี้ถูกแก้ไขแล้ว 1 ครั้ง, ครั้งล่าสุดโดยคุณ nooonuii
ตอบพร้อมอ้างอิงข้อความนี้
  #15  
Old 09 มกราคม 2005, 01:33
aaaa's Avatar
aaaa aaaa ไม่อยู่ในระบบ
ลมปราณบริสุทธิ์
 
วันที่สมัครสมาชิก: 01 มกราคม 2005
ข้อความ: 109
aaaa is on a distinguished road
Post

เฉลยข้อ 24
เพียงพอที่จะแสดงว่า
\[
\sum_{n=1}^\infty\ln(1+a_n)<\infty
\]
แต่เนื่องจาก \( \ln(1+a_n)\leq a_n\) ดังนั้นอนุกรมดังกล่าวลู่เข้า
ตอบพร้อมอ้างอิงข้อความนี้
ตั้งหัวข้อใหม่ Reply


หัวข้อคล้ายคลึงกัน
หัวข้อ ผู้ตั้งหัวข้อ ห้อง คำตอบ ข้อความล่าสุด
ข้อสอบสมาคม ม.ปลายปี 2548 prachya ข้อสอบในโรงเรียน ม.ปลาย 32 30 ตุลาคม 2010 12:58
ขอถามสสวท.2548หน่อยไม่มั่นใจ Wind ข้อสอบในโรงเรียน ประถมปลาย 3 27 สิงหาคม 2007 20:37
สมาคมคณิตศาสตร์ 2548 (ม.ต้น) R-Tummykung de Lamar ข้อสอบในโรงเรียน ม.ต้น 14 06 สิงหาคม 2006 11:03
โจทย์ปัญหาคณิตศาสตร์ สวัสดีปีใหม่ 2548 ครับ nooonuii ปัญหาคณิตศาสตร์ทั่วไป 71 08 มกราคม 2005 23:16
สสวท .เริ่มรับสมัครสอบ แข่งโอลิมปิกปี 2548 gon ข่าวคราวแวดวง ม.ปลาย 3 29 พฤษภาคม 2004 20:40


กฎการส่งข้อความ
คุณ ไม่สามารถ ตั้งหัวข้อใหม่ได้
คุณ ไม่สามารถ ตอบหัวข้อได้
คุณ ไม่สามารถ แนบไฟล์และเอกสารได้
คุณ ไม่สามารถ แก้ไขข้อความของคุณเองได้

vB code is On
Smilies are On
[IMG] code is On
HTML code is Off
ทางลัดสู่ห้อง


เวลาที่แสดงทั้งหมด เป็นเวลาที่ประเทศไทย (GMT +7) ขณะนี้เป็นเวลา 02:28


Powered by vBulletin® Copyright ©2000 - 2024, Jelsoft Enterprises Ltd.
Modified by Jetsada Karnpracha